Physics Technology Update (4th Edition)

Published by Pearson
ISBN 10: 0-32190-308-0
ISBN 13: 978-0-32190-308-2

Chapter 26 - Geometrical Optics - Problems and Conceptual Exercises - Page 946: 122

Answer

$n_1sin\theta_1=n_3sin\theta_3$

Work Step by Step

According to Snell's law $n_1sin\theta_1=n_2sin\theta_2$..........eq(1) $(n_2\gt n_1)$ Now the angle of refraction $\theta_2$ acts as an incident ray for the second medium $n_2$ and the angle of refraction becomes $\theta_3$ in medium ($n_3$) Thus $n_2sin\theta_2=n_3sin\theta_3$......eq(2) $n_3\gt n_2$ From eq(1) and eq(2), we have $n_1sin\theta_1=n_3sin\theta_3$
Update this answer!

You can help us out by revising, improving and updating this answer.

Update this answer

After you claim an answer you’ll have 24 hours to send in a draft. An editor will review the submission and either publish your submission or provide feedback.